Question

The following has 3 exercises and each exercise consists of multiple parts. please take into consideration all 3 exercises and answer each and every part of each exercise including the subquestions found in it!!

Exercise 1 Find the Laplace transform of the following functions: 1. f(t) = 5t3e-46 2. f(t) = cos(26)U(t - T) 3. k(t) = {2-1,


0 0
Add a comment Improve this question Transcribed image text
Answer #1

Date: fle) = 543 & 4t Page No. Lft) = SL § (+4)?? LHA) = 54 30 (P+4) 4 (P+4)4 fri= cos (2 st) U lart) Lift? - elp L{cos (22+26 GIS) ta (24) Li? - -9 @ Fisl - t IL & F (s) 3 = sint 1t Trt { co101 - 6 utі-ап) 9 +2t j = 8 (+) 17101=1, y polo 91+1 - 2every where replace p by s

Add a comment
Know the answer?
Add Answer to:
The following has 3 exercises and each exercise consists of multiple parts. please take into consideration...
Your Answer:

Post as a guest

Your Name:

What's your source?

Earn Coins

Coins can be redeemed for fabulous gifts.

Not the answer you're looking for? Ask your own homework help question. Our experts will answer your question WITHIN MINUTES for Free.
Similar Homework Help Questions
  • In this exercise we will use the Laplace transform to solve the following initial value problem:

    In this exercise we will use the Laplace transform to solve the following initial value problem: y"-2y'+ 17y-17, y(0)=0, y'(0)=1 (1) First, using Y for the Laplace transform of y(t), i.e., Y =L(y(t)), find the equation obtained by taking the Laplace transform of the initial value problem (2) Next solve for Y= (3) Finally apply the inverse Laplace transform to find y(t)

  • (1 point) In this exercise we will use the Laplace transform to solve the following initial...

    (1 point) In this exercise we will use the Laplace transform to solve the following initial value problem: y" + 16 16, = { 10, 0<t<1 1<t , y(0) = 3, y'(0 = 4 (1) First, using Y for the Laplace transform of y(t), i.e., Y = L(y(t)), find the equation obtained by taking the Laplace transform of the initial value problem (2) Next solve for Y = (3) Finally apply the inverse Laplace transform to find y(t) y(t) =...

  • 3. Consider the Linear Time-Invariant (LTI) system decribed by the following differential equation: dy +504 +...

    3. Consider the Linear Time-Invariant (LTI) system decribed by the following differential equation: dy +504 + 4y = u(t) dt dt where y(t) is the output of the system and u(t) is the input. This is an Initial Value Problem (IVP) with initial conditions y(0) = 0, y = 0. Also by setting u(t) = (t) an input 8(t) is given to the system, where 8(t) is the unit impulse function. a. Write a function F(s) for a function f(t)...

  • Differential Equations Project - must be completed in Maple 2018 program NEED ALL PARTS OF THE PR...

    Differential Equations Project - must be completed in Maple 2018 program NEED ALL PARTS OF THE PROJECT (A - F) In this Maple lab you learn the Maple commands for computing Laplace transforms and inverse Laplace transforms, and using them to solve initial value problems. A. Quite simply, the calling sequence for taking the Laplace transform of a function f(t) and expressing it as a function of a new variable s is laplace(f(t),t,s) . The command for computing the inverse...

  • (1 point) In this exercise we will use the Laplace transform to solve the following initial...

    (1 point) In this exercise we will use the Laplace transform to solve the following initial value problem: y-y={o. ist 1, 031<1. y(0) = 0 (1) First, using Y for the Laplace transform of y(t), i.e., Y = L(y(t)), find the equation obtained by taking the Laplace transform of the initial value problem (2) Next solve for Y = (3) Finally apply the inverse Laplace transform to find y(t) y) = (1 point) Consider the initial value problem O +6y=...

  • Consider the following initial value problem. y′ + 5y  = { 0 t  ≤  1 10...

    Consider the following initial value problem. y′ + 5y  = { 0 t  ≤  1 10 1  ≤  t  <  6 0 6  ≤  t  <  ∞ y(0)  =  4 (a) Find the Laplace transform of the right hand side of the above differential equation. (b) Let y(t) denote the solution to the above differential equation, and let Y((s) denote the Laplace transform of y(t). Find Y(s). (c) By taking the inverse Laplace transform of your answer to (b), the...

  • Hw9: Problem 12 Previous Problem Problem List Next Problem (1 point) In this exercise we will...

    Hw9: Problem 12 Previous Problem Problem List Next Problem (1 point) In this exercise we will use the Laplace transform to solve the following initial value problem: 0<, î < 1· 3, y@) :0 (1) First, using Y for the Laplace transform of y(t), ie, Y = L(y(t), find the equation obtained by taking the Laplace transform of the initial value problem (2) Next solve for Y - (3) Finally apply the inverse Laplace transform to find y(t) y(t) =

  • Hello! I need help answering these Partial Differential Equations exercises! Exercise 1 Find the general solution...

    Hello! I need help answering these Partial Differential Equations exercises! Exercise 1 Find the general solution of the cquation ury(r, y) 0 in terms of wo arbitrary functions. Exercise 2 Verify that 2c9(s)ds tcontinuously differentiable function. Hint: Here you will need to use iz' ution to the wave equation u2S, where c is a constant and g is 1's rule for differentiating an integral with respect to a parameter that a given urs n the limits of integration: b(t) F(b(t))b'...

  • STRUGGLING PLEASE HELP (1 point) Use the Laplace transform to solve the following initial value problem:...

    STRUGGLING PLEASE HELP (1 point) Use the Laplace transform to solve the following initial value problem: y" – 2y + 10y = 0 y(0) = 0, y' (O) = 3 First, using Y for the Laplace transform of y(t), i.e., Y = L{y(t)}, find the equation you get by taking the Laplace transform of the differential equation = 0 Now solve for Y(s) = By completing the square in the denominator and inverting the transform, find yt) =

  • Part B (5 points each] An initial value problem y' + 2y = f(©),y(0) = 0...

    Part B (5 points each] An initial value problem y' + 2y = f(©),y(0) = 0 is to be solved by Laplace transforms. (B-1) When f(t) is depicted in the following, show that its Laplace transform can be obtained as f(t) 4 4e F(s) = [[f(t)) = 5ż (1-es). 4 -S s V 2 0 1 2: (B-2) Show that the Laplace transform of the solution, Y(s) = Ly(0)], can be obtained as 4 4(+ 1) Y(s) = s-(s +...

ADVERTISEMENT
Free Homework Help App
Download From Google Play
Scan Your Homework
to Get Instant Free Answers
Need Online Homework Help?
Ask a Question
Get Answers For Free
Most questions answered within 3 hours.
ADVERTISEMENT
ADVERTISEMENT
ADVERTISEMENT